返回列表 发帖

og10-204

也就是11版verbal的81题

 An eyeglass manufacturer tried to boost sales for the summer quarter by offering its distributors a special discount if their orders for that quarter exceeded those for last year's summer quarter by at least 20 percentMany distributors qualified for this discountEven with much merchandise discountedsales increased enough to produce a healthy gain in net profitsThe manufacturer plans to repeat this success by offering the same sort of discount for the fall quarter

Which of the followingif truemost clearly points to a flaw in the manufacturer's plan to repeat the successful performance of the summer quarter?

(A)  In general, a distributor's orders for the summer quarter are no higher than those for the spring quarter

(B)  Along with offering special discounts to qualifying distributorsthe manufacturer increased newspaper and radio advertising in those distributors’ sales areas

(C)  The distributors most likely to qualify for the manufacturer's special discount are those whose orders were unusually low a year earlier.

(D)  The distributors who qualified for the manufacturer's special discount were free to decide how much of that discount to pass on to their own customers

(E)  The distributors’ ordering more goods in the summer quarter left them overstocked for the fall quarter

Reasoning What is the flaw the manufacturer’s plan? The plan assumes that an action that succeeded once will work a second timeWhy might the plan not work this time? If the distributors increased their orders during the summer simply because they were eager to take advantage of the discountthe result may be that they are now overstocked for the fall quarterIt sothey will not need to place orders for more goodsand the plan of continuing the discount will have 1ess chance of success now.

C  Even if the qualifying distributors only reached normal levels of salesthere is no reason to think that this success will not continue in the fall

there is no reason to think that this success will not continue in the fall

答案是e,我选的是c。

我觉得因为这些qualified distributors只是去年表现特别不好所以今年才qualified,所以今年从summer到fall,manufacturer没法repeat the success。

而且按照og对c的解释,我也可以说e里面distributers仍然会为了discount而在fall quarter继续下大量订单。

请问我这样的理解为什么不对呢?

收藏 分享

CR要紧扣原题推理,凡不在推理范围内,均是无关选项。

题目的推理是; discount计划夏季成功了->秋季也会成功。求Weaken选项:

1)C选项,什么样的distributor qualify 秋季计划与推理有关吗?所以错。

2)针对你对E的反驳,即“而且按照og对c的解释,我也可以说e里面distributers仍然会为了discount而在fall quarter继续下大量订单。请问我这样的理解为什么不对呢?”。请注意,GMAT中的weaken不是严格的充分性,即只要是一个反对点即可。Distributor的夏季库存积压了,就是对秋季discount的计划的一个不利因素,至于单单这一个不利因素是否能使整个秋季计划彻底失败,GMAT并不要求。

以上只是我个人的体会,供参考。

TOP

返回列表

站长推荐 关闭


美国top10 MBA VIP申请服务

自2003年开始提供 MBA 申请服务以来,保持着90% 以上的成功率,其中Top10 MBA服务成功率更是高达95%


查看